How to find the $n-th$ term of the series $3,7,12,18…$












1












$begingroup$


My attempt:




Let,$$space space space space space space space S=3+7+12+18+......+t_n$$
$$space space space space space space space spacespace space space space space space space space space S=space space space space space space space space 3+7+12+18+...t_{n-1}+t_n$$




Subtracting,
$$0=3+4+5+6+...(t_n-t_{n-1})-t_n$$
$$implies t_n=frac{n(n+1)}{2}-3$$
By adding and subtracting $1,2$.
However this gives a negative value of $t_1$. Where is it wrong?










share|cite|improve this question









$endgroup$












  • $begingroup$
    I've never seen this approach to finding the nth term of a sequence.. I'm curious to see what others have to say.
    $endgroup$
    – T. Fo
    Jan 8 at 17:15
















1












$begingroup$


My attempt:




Let,$$space space space space space space space S=3+7+12+18+......+t_n$$
$$space space space space space space space spacespace space space space space space space space space S=space space space space space space space space 3+7+12+18+...t_{n-1}+t_n$$




Subtracting,
$$0=3+4+5+6+...(t_n-t_{n-1})-t_n$$
$$implies t_n=frac{n(n+1)}{2}-3$$
By adding and subtracting $1,2$.
However this gives a negative value of $t_1$. Where is it wrong?










share|cite|improve this question









$endgroup$












  • $begingroup$
    I've never seen this approach to finding the nth term of a sequence.. I'm curious to see what others have to say.
    $endgroup$
    – T. Fo
    Jan 8 at 17:15














1












1








1





$begingroup$


My attempt:




Let,$$space space space space space space space S=3+7+12+18+......+t_n$$
$$space space space space space space space spacespace space space space space space space space space S=space space space space space space space space 3+7+12+18+...t_{n-1}+t_n$$




Subtracting,
$$0=3+4+5+6+...(t_n-t_{n-1})-t_n$$
$$implies t_n=frac{n(n+1)}{2}-3$$
By adding and subtracting $1,2$.
However this gives a negative value of $t_1$. Where is it wrong?










share|cite|improve this question









$endgroup$




My attempt:




Let,$$space space space space space space space S=3+7+12+18+......+t_n$$
$$space space space space space space space spacespace space space space space space space space space S=space space space space space space space space 3+7+12+18+...t_{n-1}+t_n$$




Subtracting,
$$0=3+4+5+6+...(t_n-t_{n-1})-t_n$$
$$implies t_n=frac{n(n+1)}{2}-3$$
By adding and subtracting $1,2$.
However this gives a negative value of $t_1$. Where is it wrong?







sequences-and-series






share|cite|improve this question













share|cite|improve this question











share|cite|improve this question




share|cite|improve this question










asked Jan 8 at 17:12









tatantatan

5,63362655




5,63362655












  • $begingroup$
    I've never seen this approach to finding the nth term of a sequence.. I'm curious to see what others have to say.
    $endgroup$
    – T. Fo
    Jan 8 at 17:15


















  • $begingroup$
    I've never seen this approach to finding the nth term of a sequence.. I'm curious to see what others have to say.
    $endgroup$
    – T. Fo
    Jan 8 at 17:15
















$begingroup$
I've never seen this approach to finding the nth term of a sequence.. I'm curious to see what others have to say.
$endgroup$
– T. Fo
Jan 8 at 17:15




$begingroup$
I've never seen this approach to finding the nth term of a sequence.. I'm curious to see what others have to say.
$endgroup$
– T. Fo
Jan 8 at 17:15










5 Answers
5






active

oldest

votes


















2












$begingroup$

$dfrac{n^2+n}2-3=3iff0=(n+4)(n-3)$



So, your sequence start with $n=3$






share|cite|improve this answer









$endgroup$













  • $begingroup$
    Suppose I have to find the sum $S$ as given in the question. How do I do it then?
    $endgroup$
    – tatan
    Jan 8 at 17:21










  • $begingroup$
    @tatan, Use brilliant.org/wiki/sum-of-n-n2-or-n3
    $endgroup$
    – lab bhattacharjee
    Jan 8 at 17:23






  • 1




    $begingroup$
    @tatan: you did just fine, but note that $3$ is then the third term of your sum because you put $1,2$ in front. Then the last term is $t_{n+2}$. If you want to start counting your terms at $1$, you must update your formula to be $frac {(n+2)(n+3)}2-3$, which works fine.
    $endgroup$
    – Ross Millikan
    Jan 8 at 17:28










  • $begingroup$
    @RossMillikan Ahhh.... Thanks a lot ;-)
    $endgroup$
    – tatan
    Jan 8 at 17:29



















1












$begingroup$

$S= 3+7+12+18+...;$



$S= 3 +(3+4) +( 3+4+5) +(3+4+5+6) +...;$



$S= a_1 +a_2 +a_3 +a_4.......+ a_n;$



$a_n =(3+4+5+ ...(n+2));$



$a_n = 2n+(1+2+3...+n)$, or



$a_n= 2n+n(n+1)/2.$






share|cite|improve this answer









$endgroup$





















    0












    $begingroup$

    The sum of $n$ consecutive integers, from $3$ and up to $t_n- t_{n-1}$, isn't $frac{n(n+1)}2$. If you think about which (and how many) integers $frac{n(n+1)}2$ usually represents the sum of, you will hopefully see what went wrong.






    share|cite|improve this answer









    $endgroup$













    • $begingroup$
      i did make it $(1+2)+3+...(t_n-t_{n-1})-(1+2)$
      $endgroup$
      – tatan
      Jan 8 at 17:20












    • $begingroup$
      @tatan I know. I'm trying to tell you that that's not $frac{n(n+1)}2-3$.
      $endgroup$
      – Arthur
      Jan 8 at 18:38





















    0












    $begingroup$

    The systematic way is to use Newton's interpolation formula based on
    repeated differences:
    $$
    begin{array}{llll}
    3 & 7 & 12 & 18 & \
    4 & 5 & 6 & \
    1 & 1 & \
    0 & \
    end{array}
    $$

    and so
    $$
    t_n=3 binom{n-1}{0} + 4 binom{n-1}{1} + 1 binom{n-1}{2}
    $$

    which simplifies to
    $$
    t_n=dfrac{n (n + 5)}{2}
    $$






    share|cite|improve this answer









    $endgroup$





















      0












      $begingroup$

      You can prove by induction that $a_{n}=3n+cfrac{n(n-1)}{2}$

      Here, you don't have to assume that $n>2$






      share|cite|improve this answer











      $endgroup$













        Your Answer





        StackExchange.ifUsing("editor", function () {
        return StackExchange.using("mathjaxEditing", function () {
        StackExchange.MarkdownEditor.creationCallbacks.add(function (editor, postfix) {
        StackExchange.mathjaxEditing.prepareWmdForMathJax(editor, postfix, [["$", "$"], ["\\(","\\)"]]);
        });
        });
        }, "mathjax-editing");

        StackExchange.ready(function() {
        var channelOptions = {
        tags: "".split(" "),
        id: "69"
        };
        initTagRenderer("".split(" "), "".split(" "), channelOptions);

        StackExchange.using("externalEditor", function() {
        // Have to fire editor after snippets, if snippets enabled
        if (StackExchange.settings.snippets.snippetsEnabled) {
        StackExchange.using("snippets", function() {
        createEditor();
        });
        }
        else {
        createEditor();
        }
        });

        function createEditor() {
        StackExchange.prepareEditor({
        heartbeatType: 'answer',
        autoActivateHeartbeat: false,
        convertImagesToLinks: true,
        noModals: true,
        showLowRepImageUploadWarning: true,
        reputationToPostImages: 10,
        bindNavPrevention: true,
        postfix: "",
        imageUploader: {
        brandingHtml: "Powered by u003ca class="icon-imgur-white" href="https://imgur.com/"u003eu003c/au003e",
        contentPolicyHtml: "User contributions licensed under u003ca href="https://creativecommons.org/licenses/by-sa/3.0/"u003ecc by-sa 3.0 with attribution requiredu003c/au003e u003ca href="https://stackoverflow.com/legal/content-policy"u003e(content policy)u003c/au003e",
        allowUrls: true
        },
        noCode: true, onDemand: true,
        discardSelector: ".discard-answer"
        ,immediatelyShowMarkdownHelp:true
        });


        }
        });














        draft saved

        draft discarded


















        StackExchange.ready(
        function () {
        StackExchange.openid.initPostLogin('.new-post-login', 'https%3a%2f%2fmath.stackexchange.com%2fquestions%2f3066451%2fhow-to-find-the-n-th-term-of-the-series-3-7-12-18%23new-answer', 'question_page');
        }
        );

        Post as a guest















        Required, but never shown

























        5 Answers
        5






        active

        oldest

        votes








        5 Answers
        5






        active

        oldest

        votes









        active

        oldest

        votes






        active

        oldest

        votes









        2












        $begingroup$

        $dfrac{n^2+n}2-3=3iff0=(n+4)(n-3)$



        So, your sequence start with $n=3$






        share|cite|improve this answer









        $endgroup$













        • $begingroup$
          Suppose I have to find the sum $S$ as given in the question. How do I do it then?
          $endgroup$
          – tatan
          Jan 8 at 17:21










        • $begingroup$
          @tatan, Use brilliant.org/wiki/sum-of-n-n2-or-n3
          $endgroup$
          – lab bhattacharjee
          Jan 8 at 17:23






        • 1




          $begingroup$
          @tatan: you did just fine, but note that $3$ is then the third term of your sum because you put $1,2$ in front. Then the last term is $t_{n+2}$. If you want to start counting your terms at $1$, you must update your formula to be $frac {(n+2)(n+3)}2-3$, which works fine.
          $endgroup$
          – Ross Millikan
          Jan 8 at 17:28










        • $begingroup$
          @RossMillikan Ahhh.... Thanks a lot ;-)
          $endgroup$
          – tatan
          Jan 8 at 17:29
















        2












        $begingroup$

        $dfrac{n^2+n}2-3=3iff0=(n+4)(n-3)$



        So, your sequence start with $n=3$






        share|cite|improve this answer









        $endgroup$













        • $begingroup$
          Suppose I have to find the sum $S$ as given in the question. How do I do it then?
          $endgroup$
          – tatan
          Jan 8 at 17:21










        • $begingroup$
          @tatan, Use brilliant.org/wiki/sum-of-n-n2-or-n3
          $endgroup$
          – lab bhattacharjee
          Jan 8 at 17:23






        • 1




          $begingroup$
          @tatan: you did just fine, but note that $3$ is then the third term of your sum because you put $1,2$ in front. Then the last term is $t_{n+2}$. If you want to start counting your terms at $1$, you must update your formula to be $frac {(n+2)(n+3)}2-3$, which works fine.
          $endgroup$
          – Ross Millikan
          Jan 8 at 17:28










        • $begingroup$
          @RossMillikan Ahhh.... Thanks a lot ;-)
          $endgroup$
          – tatan
          Jan 8 at 17:29














        2












        2








        2





        $begingroup$

        $dfrac{n^2+n}2-3=3iff0=(n+4)(n-3)$



        So, your sequence start with $n=3$






        share|cite|improve this answer









        $endgroup$



        $dfrac{n^2+n}2-3=3iff0=(n+4)(n-3)$



        So, your sequence start with $n=3$







        share|cite|improve this answer












        share|cite|improve this answer



        share|cite|improve this answer










        answered Jan 8 at 17:16









        lab bhattacharjeelab bhattacharjee

        225k15156274




        225k15156274












        • $begingroup$
          Suppose I have to find the sum $S$ as given in the question. How do I do it then?
          $endgroup$
          – tatan
          Jan 8 at 17:21










        • $begingroup$
          @tatan, Use brilliant.org/wiki/sum-of-n-n2-or-n3
          $endgroup$
          – lab bhattacharjee
          Jan 8 at 17:23






        • 1




          $begingroup$
          @tatan: you did just fine, but note that $3$ is then the third term of your sum because you put $1,2$ in front. Then the last term is $t_{n+2}$. If you want to start counting your terms at $1$, you must update your formula to be $frac {(n+2)(n+3)}2-3$, which works fine.
          $endgroup$
          – Ross Millikan
          Jan 8 at 17:28










        • $begingroup$
          @RossMillikan Ahhh.... Thanks a lot ;-)
          $endgroup$
          – tatan
          Jan 8 at 17:29


















        • $begingroup$
          Suppose I have to find the sum $S$ as given in the question. How do I do it then?
          $endgroup$
          – tatan
          Jan 8 at 17:21










        • $begingroup$
          @tatan, Use brilliant.org/wiki/sum-of-n-n2-or-n3
          $endgroup$
          – lab bhattacharjee
          Jan 8 at 17:23






        • 1




          $begingroup$
          @tatan: you did just fine, but note that $3$ is then the third term of your sum because you put $1,2$ in front. Then the last term is $t_{n+2}$. If you want to start counting your terms at $1$, you must update your formula to be $frac {(n+2)(n+3)}2-3$, which works fine.
          $endgroup$
          – Ross Millikan
          Jan 8 at 17:28










        • $begingroup$
          @RossMillikan Ahhh.... Thanks a lot ;-)
          $endgroup$
          – tatan
          Jan 8 at 17:29
















        $begingroup$
        Suppose I have to find the sum $S$ as given in the question. How do I do it then?
        $endgroup$
        – tatan
        Jan 8 at 17:21




        $begingroup$
        Suppose I have to find the sum $S$ as given in the question. How do I do it then?
        $endgroup$
        – tatan
        Jan 8 at 17:21












        $begingroup$
        @tatan, Use brilliant.org/wiki/sum-of-n-n2-or-n3
        $endgroup$
        – lab bhattacharjee
        Jan 8 at 17:23




        $begingroup$
        @tatan, Use brilliant.org/wiki/sum-of-n-n2-or-n3
        $endgroup$
        – lab bhattacharjee
        Jan 8 at 17:23




        1




        1




        $begingroup$
        @tatan: you did just fine, but note that $3$ is then the third term of your sum because you put $1,2$ in front. Then the last term is $t_{n+2}$. If you want to start counting your terms at $1$, you must update your formula to be $frac {(n+2)(n+3)}2-3$, which works fine.
        $endgroup$
        – Ross Millikan
        Jan 8 at 17:28




        $begingroup$
        @tatan: you did just fine, but note that $3$ is then the third term of your sum because you put $1,2$ in front. Then the last term is $t_{n+2}$. If you want to start counting your terms at $1$, you must update your formula to be $frac {(n+2)(n+3)}2-3$, which works fine.
        $endgroup$
        – Ross Millikan
        Jan 8 at 17:28












        $begingroup$
        @RossMillikan Ahhh.... Thanks a lot ;-)
        $endgroup$
        – tatan
        Jan 8 at 17:29




        $begingroup$
        @RossMillikan Ahhh.... Thanks a lot ;-)
        $endgroup$
        – tatan
        Jan 8 at 17:29











        1












        $begingroup$

        $S= 3+7+12+18+...;$



        $S= 3 +(3+4) +( 3+4+5) +(3+4+5+6) +...;$



        $S= a_1 +a_2 +a_3 +a_4.......+ a_n;$



        $a_n =(3+4+5+ ...(n+2));$



        $a_n = 2n+(1+2+3...+n)$, or



        $a_n= 2n+n(n+1)/2.$






        share|cite|improve this answer









        $endgroup$


















          1












          $begingroup$

          $S= 3+7+12+18+...;$



          $S= 3 +(3+4) +( 3+4+5) +(3+4+5+6) +...;$



          $S= a_1 +a_2 +a_3 +a_4.......+ a_n;$



          $a_n =(3+4+5+ ...(n+2));$



          $a_n = 2n+(1+2+3...+n)$, or



          $a_n= 2n+n(n+1)/2.$






          share|cite|improve this answer









          $endgroup$
















            1












            1








            1





            $begingroup$

            $S= 3+7+12+18+...;$



            $S= 3 +(3+4) +( 3+4+5) +(3+4+5+6) +...;$



            $S= a_1 +a_2 +a_3 +a_4.......+ a_n;$



            $a_n =(3+4+5+ ...(n+2));$



            $a_n = 2n+(1+2+3...+n)$, or



            $a_n= 2n+n(n+1)/2.$






            share|cite|improve this answer









            $endgroup$



            $S= 3+7+12+18+...;$



            $S= 3 +(3+4) +( 3+4+5) +(3+4+5+6) +...;$



            $S= a_1 +a_2 +a_3 +a_4.......+ a_n;$



            $a_n =(3+4+5+ ...(n+2));$



            $a_n = 2n+(1+2+3...+n)$, or



            $a_n= 2n+n(n+1)/2.$







            share|cite|improve this answer












            share|cite|improve this answer



            share|cite|improve this answer










            answered Jan 8 at 18:05









            Peter SzilasPeter Szilas

            11.1k2821




            11.1k2821























                0












                $begingroup$

                The sum of $n$ consecutive integers, from $3$ and up to $t_n- t_{n-1}$, isn't $frac{n(n+1)}2$. If you think about which (and how many) integers $frac{n(n+1)}2$ usually represents the sum of, you will hopefully see what went wrong.






                share|cite|improve this answer









                $endgroup$













                • $begingroup$
                  i did make it $(1+2)+3+...(t_n-t_{n-1})-(1+2)$
                  $endgroup$
                  – tatan
                  Jan 8 at 17:20












                • $begingroup$
                  @tatan I know. I'm trying to tell you that that's not $frac{n(n+1)}2-3$.
                  $endgroup$
                  – Arthur
                  Jan 8 at 18:38


















                0












                $begingroup$

                The sum of $n$ consecutive integers, from $3$ and up to $t_n- t_{n-1}$, isn't $frac{n(n+1)}2$. If you think about which (and how many) integers $frac{n(n+1)}2$ usually represents the sum of, you will hopefully see what went wrong.






                share|cite|improve this answer









                $endgroup$













                • $begingroup$
                  i did make it $(1+2)+3+...(t_n-t_{n-1})-(1+2)$
                  $endgroup$
                  – tatan
                  Jan 8 at 17:20












                • $begingroup$
                  @tatan I know. I'm trying to tell you that that's not $frac{n(n+1)}2-3$.
                  $endgroup$
                  – Arthur
                  Jan 8 at 18:38
















                0












                0








                0





                $begingroup$

                The sum of $n$ consecutive integers, from $3$ and up to $t_n- t_{n-1}$, isn't $frac{n(n+1)}2$. If you think about which (and how many) integers $frac{n(n+1)}2$ usually represents the sum of, you will hopefully see what went wrong.






                share|cite|improve this answer









                $endgroup$



                The sum of $n$ consecutive integers, from $3$ and up to $t_n- t_{n-1}$, isn't $frac{n(n+1)}2$. If you think about which (and how many) integers $frac{n(n+1)}2$ usually represents the sum of, you will hopefully see what went wrong.







                share|cite|improve this answer












                share|cite|improve this answer



                share|cite|improve this answer










                answered Jan 8 at 17:19









                ArthurArthur

                113k7110193




                113k7110193












                • $begingroup$
                  i did make it $(1+2)+3+...(t_n-t_{n-1})-(1+2)$
                  $endgroup$
                  – tatan
                  Jan 8 at 17:20












                • $begingroup$
                  @tatan I know. I'm trying to tell you that that's not $frac{n(n+1)}2-3$.
                  $endgroup$
                  – Arthur
                  Jan 8 at 18:38




















                • $begingroup$
                  i did make it $(1+2)+3+...(t_n-t_{n-1})-(1+2)$
                  $endgroup$
                  – tatan
                  Jan 8 at 17:20












                • $begingroup$
                  @tatan I know. I'm trying to tell you that that's not $frac{n(n+1)}2-3$.
                  $endgroup$
                  – Arthur
                  Jan 8 at 18:38


















                $begingroup$
                i did make it $(1+2)+3+...(t_n-t_{n-1})-(1+2)$
                $endgroup$
                – tatan
                Jan 8 at 17:20






                $begingroup$
                i did make it $(1+2)+3+...(t_n-t_{n-1})-(1+2)$
                $endgroup$
                – tatan
                Jan 8 at 17:20














                $begingroup$
                @tatan I know. I'm trying to tell you that that's not $frac{n(n+1)}2-3$.
                $endgroup$
                – Arthur
                Jan 8 at 18:38






                $begingroup$
                @tatan I know. I'm trying to tell you that that's not $frac{n(n+1)}2-3$.
                $endgroup$
                – Arthur
                Jan 8 at 18:38













                0












                $begingroup$

                The systematic way is to use Newton's interpolation formula based on
                repeated differences:
                $$
                begin{array}{llll}
                3 & 7 & 12 & 18 & \
                4 & 5 & 6 & \
                1 & 1 & \
                0 & \
                end{array}
                $$

                and so
                $$
                t_n=3 binom{n-1}{0} + 4 binom{n-1}{1} + 1 binom{n-1}{2}
                $$

                which simplifies to
                $$
                t_n=dfrac{n (n + 5)}{2}
                $$






                share|cite|improve this answer









                $endgroup$


















                  0












                  $begingroup$

                  The systematic way is to use Newton's interpolation formula based on
                  repeated differences:
                  $$
                  begin{array}{llll}
                  3 & 7 & 12 & 18 & \
                  4 & 5 & 6 & \
                  1 & 1 & \
                  0 & \
                  end{array}
                  $$

                  and so
                  $$
                  t_n=3 binom{n-1}{0} + 4 binom{n-1}{1} + 1 binom{n-1}{2}
                  $$

                  which simplifies to
                  $$
                  t_n=dfrac{n (n + 5)}{2}
                  $$






                  share|cite|improve this answer









                  $endgroup$
















                    0












                    0








                    0





                    $begingroup$

                    The systematic way is to use Newton's interpolation formula based on
                    repeated differences:
                    $$
                    begin{array}{llll}
                    3 & 7 & 12 & 18 & \
                    4 & 5 & 6 & \
                    1 & 1 & \
                    0 & \
                    end{array}
                    $$

                    and so
                    $$
                    t_n=3 binom{n-1}{0} + 4 binom{n-1}{1} + 1 binom{n-1}{2}
                    $$

                    which simplifies to
                    $$
                    t_n=dfrac{n (n + 5)}{2}
                    $$






                    share|cite|improve this answer









                    $endgroup$



                    The systematic way is to use Newton's interpolation formula based on
                    repeated differences:
                    $$
                    begin{array}{llll}
                    3 & 7 & 12 & 18 & \
                    4 & 5 & 6 & \
                    1 & 1 & \
                    0 & \
                    end{array}
                    $$

                    and so
                    $$
                    t_n=3 binom{n-1}{0} + 4 binom{n-1}{1} + 1 binom{n-1}{2}
                    $$

                    which simplifies to
                    $$
                    t_n=dfrac{n (n + 5)}{2}
                    $$







                    share|cite|improve this answer












                    share|cite|improve this answer



                    share|cite|improve this answer










                    answered Jan 8 at 17:25









                    lhflhf

                    164k10170395




                    164k10170395























                        0












                        $begingroup$

                        You can prove by induction that $a_{n}=3n+cfrac{n(n-1)}{2}$

                        Here, you don't have to assume that $n>2$






                        share|cite|improve this answer











                        $endgroup$


















                          0












                          $begingroup$

                          You can prove by induction that $a_{n}=3n+cfrac{n(n-1)}{2}$

                          Here, you don't have to assume that $n>2$






                          share|cite|improve this answer











                          $endgroup$
















                            0












                            0








                            0





                            $begingroup$

                            You can prove by induction that $a_{n}=3n+cfrac{n(n-1)}{2}$

                            Here, you don't have to assume that $n>2$






                            share|cite|improve this answer











                            $endgroup$



                            You can prove by induction that $a_{n}=3n+cfrac{n(n-1)}{2}$

                            Here, you don't have to assume that $n>2$







                            share|cite|improve this answer














                            share|cite|improve this answer



                            share|cite|improve this answer








                            edited Jan 8 at 17:58

























                            answered Jan 8 at 17:19









                            The CatThe Cat

                            18611




                            18611






























                                draft saved

                                draft discarded




















































                                Thanks for contributing an answer to Mathematics Stack Exchange!


                                • Please be sure to answer the question. Provide details and share your research!

                                But avoid



                                • Asking for help, clarification, or responding to other answers.

                                • Making statements based on opinion; back them up with references or personal experience.


                                Use MathJax to format equations. MathJax reference.


                                To learn more, see our tips on writing great answers.




                                draft saved


                                draft discarded














                                StackExchange.ready(
                                function () {
                                StackExchange.openid.initPostLogin('.new-post-login', 'https%3a%2f%2fmath.stackexchange.com%2fquestions%2f3066451%2fhow-to-find-the-n-th-term-of-the-series-3-7-12-18%23new-answer', 'question_page');
                                }
                                );

                                Post as a guest















                                Required, but never shown





















































                                Required, but never shown














                                Required, but never shown












                                Required, but never shown







                                Required, but never shown

































                                Required, but never shown














                                Required, but never shown












                                Required, but never shown







                                Required, but never shown







                                Popular posts from this blog

                                An IMO inspired problem

                                Management

                                Has there ever been an instance of an active nuclear power plant within or near a war zone?